高級宏觀經濟學-第四版-中文-羅默課后題答案_第1頁
高級宏觀經濟學-第四版-中文-羅默課后題答案_第2頁
高級宏觀經濟學-第四版-中文-羅默課后題答案_第3頁
高級宏觀經濟學-第四版-中文-羅默課后題答案_第4頁
高級宏觀經濟學-第四版-中文-羅默課后題答案_第5頁
已閱讀5頁,還剩48頁未讀 繼續(xù)免費閱讀

下載本文檔

版權說明:本文檔由用戶提供并上傳,收益歸屬內容提供方,若內容存在侵權,請進行舉報或認領

文檔簡介

1、精選優(yōu)質文檔-傾情為你奉上精選優(yōu)質文檔-傾情為你奉上專心-專注-專業(yè)專心-專注-專業(yè)精選優(yōu)質文檔-傾情為你奉上專心-專注-專業(yè)高級宏觀經濟學_第四版_中文_羅默課后題答案ADDIN CNKISM.UserStyle第2章無限期模型與世代交疊模型2.1 考慮N個廠商,每個廠商均有規(guī)模報酬不變的生產函數Y=FK,AL,,或者采用緊湊形式Y=ALfk。假設f0,f0,則積分項收斂,為1-n,則:-1=WL(0)H-n(7)將方程(7)代入(4):Ct=eR(t)-tWL(0)H-n(8)因此,初始消費為:C0=WL(0)H-n(9)個人的初始財富為WL(0)H,方程(9)說明消費是初始財富的一個不變

2、的比例。-n為個人的財富邊際消費傾向??梢钥闯?,這個財富邊際消費傾向在平衡增長路徑上是獨立于利率的。對于折現率而言,越大,家庭越厭惡風險,越會選擇多消費。2.5 設想某家庭的效用函數由(2.1)(2.2)式給定。假設實際利率不變,令W表示家庭的初始財富加上終生勞動收入的現值(2.6)的右端。已知r、W和效用函數中的各參數,求C的效用最大化路徑。U=t=0e-tuCtL(t)Hdt2.1uCt=Ct1-1-2.2答:本題目是在家庭的預算約束下最大化一生的效用,即:maxU=t=0e-tuCtL(t)Hdt(1)s.t. t=0e-rtCtL(t)Hdt=W(2)W代表家庭的初始財富加上家庭一生勞

3、動收入的現值,利率r是常數。建立拉格朗日方程如下:L=t=0e-tCt1-1-L(t)Hdt+W-t=0e-rtCtL(t)Hdt求一階條件,可得:LCt=e-tCt-L(t)H-e-rtLtH=0抵消L(t)/H,得:e-tCt-=e-rt(3)兩邊對時間t求導,可得:e-t-Ct-1Ct-e-tCt-+re-rt=0得到下面的方程:-CtCte-tCt-e-tCt-+re-rt=0(4)將方程(3)代入(4),可得:-CtCt e-rt-e-rt+re-rt=0抵消e-rt然后求消費的增長率CtCt,可得:CtCt=r-(5)由于利率r是常數,所以消費的增長率為常數。如果r,則市場利率超過

4、貼現率,則消費會增加;反之,如果r,則決定了消費增長的幅度。值越低,也就是替代彈性越高,1越高,即消費增長的越快。重寫方程(5),得:lnCtt=r-(6)對方程(6)積分,積分區(qū)間是從時間=0到時間=t,可得:lnCt-lnC0=r-=0=t上式可以簡化為:lnCtC0=(r-)/t(7)對方程(7)兩邊取指數,可得:CtC0=e(r-)/t,整理得:Ct=C0e(r-)/t(8)下面求解初始消費,將方程(8)代入(2),可得:t=0e-rtC0e(r-)/tL(t)Hdt=W將Lt=entL0代入上式,可得:C0L(0)Ht=0e-r+r-nt/dt=W(9)只要-r+r-n/0,從而保證

5、積分收斂,則求解方程(9)可得:t=0e-r+r-nt/dt=-r+r-n(10)將方程(10)代入(9)中,求解C0:C0=WL0H-r+r-n(11)將方程(11)代入(8),求解Ct:Ct=e(r-)/tWL0H-r+r-n(12)上式便是C的效用最大化路徑。2.6 生產力增長減速與儲蓄。設想一個正處于平衡增長路徑上的拉姆塞卡斯庫普曼期模型,假設g永久性下降。(a)k=0曲線會如何變化(如果有影響)?(b)c=0曲線會如何變化(如果有影響)?(c)當g下降時,c如何變化?(d)用一個式子表示g的邊際變化對平衡增長路徑上儲蓄率的影響。能否判斷此表達式的正負?(e)設生產函數是柯布道格拉斯函

6、數fk=k,請用、n、g、和重新表示(d)中的結果。(提示:利用等式fk*=+g。)答:(a)關于資本的歐拉方程為:kt=fkt-ct-n+gkt(1)該方程描述了資本的動態(tài)方程,在拉姆塞模型中,該方程描述了技術特征,是該模型的核心,它與消費的動態(tài)方程一起構成了該模型的歐拉方程組,從而決定了該模型的最終解。圖2-1 拉姆塞模型在平衡增長路徑上,k=0,由此可以推出:c=fk-n+gk。在該方程中,當g永久性地下降時,會導致消費c上升以保持方程的均衡。因而在圖形上k=0曲線向上移動。同時,保持k不變,g永久性地下降會導致持平投資下降,這樣就會有更多的資源用于消費。由于持平投資n+g下降的幅度更大

7、,因而在更高的k水平上,k=0向上移動得更大。圖2-1是該模型的圖示。(b)每單位有效勞動消費的歐拉方程為:ctct=fkt-g(2)該方程描述了消費的動態(tài)方程,在拉姆塞模型中,該方程描述了偏好特征,是該模型的核心,它與資本的動態(tài)方程一起構成了該模型的歐拉方程組,從而決定了該模型的最終解。在平衡增長路徑上,要求c=0,即fk=+g,在g永久性地下降時,為保持c=0,fk必須下降。由于fk0,因而fk下降必然導致k上升。因此,c=0必須上升,在圖形上表現為c=0向右移動,如圖2-1所示。(c)在g永久性地下降時,由于每單位有效勞動的資本是由歷史上的投資決定的,因而不會發(fā)生不連續(xù)的變化。它仍然保持

8、在平衡增長路徑k*處。與此相反,每單位有效勞動的消費則會隨著g永久性地下降而迅速變化。為使經濟從舊的平衡增長路徑達到新的平衡增長路徑,每單位有效勞動的消費c必將發(fā)生變化。不過,此處無法確定新的平衡增長路徑處于舊的均衡點的上邊還是下邊,因而無法確定每單位有效勞動的消費c是上升還是下降。存在一種特殊情況,即如果新的平衡增長路徑恰好位于舊的均衡點的右上方,則每單位有效勞動的消費c甚至可能保持不變。因此,c和k逐步移動到新的平衡增長路徑,此時的值高于原先的平衡增長路徑值。(d)在平衡增長路徑上,產出中被儲蓄的部分為:fk*-c*fk*因為k保持不變,即k=0,位于一條均衡的增長路徑上,則由方程(1)可

9、知:fk*-c*=n+gk*由上面兩個式子可以推出在平衡增長路徑上,產出中被儲蓄的份額為:s=n+gk*fk*(3)對方程(3)兩邊關于g求導數,可得:sg=fk*n+gk*g+k*-n+gk*fk*k*gfk*2可以再簡化為:sg=n+gfk*-k*fk*k*g+fk*k*fk*2(4)由于k*由fk=+g決定,對該式兩邊關于g求導數,可得:fk*k*g=,從而求出k*g為:k*g=fk*0(5)將方程(5)代入(4)中,可得:sg=n+gfk*-k*fk*+fk*k*fk*fk*2fk*(6)在方程(6)中,分母fk*2fk*為負,分子中第一項為正,而第二項為負,因而無法確定正與負。因此,

10、無法判斷在平衡增長路徑上g永久性地下降會使s上升還是下降。(e)將柯布道格拉斯生產函數fk=k,fk=k-1和fk=-1k-2代入方程(6)中,可得:sg=n+gk*-k*k*-1+k*k*-1k*-2k*k*-1k*-2簡化為:sg=n+gk*1-1-k*k*-1-1-k*k*-1k*-1從上式可以推出:sg=-n+g-+g+g2最終有下面的結果:sg=-n-+g2=-n+g22.7 說明下列變化如何影響圖2.5中的c=0線和k=0線,并在此基礎上說明其如何影響平衡增長路徑上的c值和k值。(a)上升(b)生產函數向下移動。(c)折舊率由本章中假設的零變?yōu)槟骋徽怠D2-2 鞍點路徑答:(a)

11、關于c與k的歐拉方程為:ctct=fkt-g(1)kt=fkt-ct-n+gkt(2)的上升即消費的跨期替代彈性1/下降,表明家庭不太愿意接受消費的跨期替代,同時表明隨著消費的上升,消費的邊際產品下降得很快。這種情況使家庭更偏好于即期消費。由于沒有出現在資本積累方程(2)中,因而資本積累方程不受的上升的影響。在消費的動態(tài)方程中,在平衡增長路徑上c=0,從而fk=-g,由于的上升,因而fk必須上升,又因為fk0的情況時,消費和資本的歐拉方程變?yōu)椋篶tct=fkt-g(1)kt=fkt-ct-n+g+kt(2)對方程(1)和(2)分別在c=c*和k=k*處進行一階泰勒展開,可得:c=ckk-k*+

12、ccc-c*(3)k=kkk-k*+kcc-c*(4)定義c=c-c*和k=k-k*,因為c*和k*為常數,所以c=c且k=k,將(3)和(4)重寫為:c=ckk+ccc(5)k=kkk+kcc(6)對方程(1)和(2)計算偏導數:ckbgp=fk*c*(7)ccbgp=fk*-g(8)kkbgp=fk*-n+g+(9)kcbgp=-1(10)將方程(7)和(8)代入(5),將方程(9)和(10)代入(6),可得:c=fk*c*k(11)k=fk*-n+g+k-c =+g-n+g+k-c =k-c(12)方程(12)的第二步用到了fk*=+g,第三步用到了定義=-n-1-g。對方程(11)除以

13、c以求c的增長率,對方程(12)除以k以求k的增長率:cc=fk*c*kc(13)kk=-ck(14)可以發(fā)現該結果與教材中不存在折舊率的增長率一樣,也就是說折舊率的存在對增長率沒有影響。因此,經濟在向平衡增長路徑移動時的c和k的不變增長率與教材中的結果應該一致。令=cc方程(13)可以推出:ck=fk*c*1(15)由方程(15),令(13)和(14)相等,可得:=-fk*c*1,求解可得:=2-4fk*c*122如果為正,則經濟會偏離穩(wěn)定點,所以必為負:1=-2-4fk*c*122現在考慮柯布道格拉斯生產函數fk=k,分別求其一階導和二階導:fk*=k*-1=r*+(16)fk*=k*-1

14、=-1k*-2(17)將方程(16)兩邊同時平方:r*+2=2k*2-2,將其代入(17)式:fk*=r*+2-1k*=-1r*+2fk*定義平衡增長路徑上的儲蓄率為s*,則平衡增長路徑上的消費為:c*=1-s*fk*(18)將方程(17)和(18)代入(15):1=-2-4-1r*+2fk*1-s*fk*2化簡為:1=-2+41-r*+21-s*2(19)在平衡增長路徑上,c=0意味著r*=+g,即:r*+=+g+(20)另外,實際投資等于持平投資:s*fk*=n+g+k*,可以推出:s*=n+g+k*fk*=n+g+k*-1(21)上步用到了r*+=k*-1,由(21)可以推出:1-s*=

15、r*+-n+g+r*+(22)將方程(20)和(22)代入到(19)中,可得:1=-2+41-+g+g+-n+g+2上式與教材中的(2.39)極其相似,它表明了消費與資本的調整速度(將=13,=4%,n=2%,g=1%,=1,=3%代入上式,得到1=-8.8%)要快于不存在折舊時的調整速度。2.9 拉姆塞模型的解析解來自于史密斯(Smith,2006)??紤]生產函數柯布-道格拉斯函數的拉姆塞模型,yt=k(t)的情形,假設相對風險規(guī)避系數與資本份額相等。(a)平衡增長路徑上的k值(即k*)為多少?(b)平衡增長路徑上的c值(即c*)為多少?(c)令z(t)表示資本產出比k(t)y(t),x(t

16、)表示消費資本比c(t)k(t)。請用z、x和模型參數表示zt和xtx(t)。(d)暫且猜測x在鞍點路徑上是常數,根據這一猜想:(i)給定初始值z(0),求z的路徑。(ii)給定初始值k(0),求y的路徑。經濟沿鞍點路徑向平衡增長路徑收斂的速度是否是常數?(e)上述猜測的解是否滿足c與k的運動方程(2.24)與(2.25)?答:(a)已知yt=k(t)(1)從正文可知,在c=0時,存在fk=+g。利用方程(1)計算得到k*=+g11-(2)(b)與(a)題類似,根據正文可知,在k=0時,存在c*=fk-n+gk。利用方程(1)計算得到:c*=+g1-n+g+g11-(3)(c)設zt=k(t)

17、y(t)和xt=c(t)k(t)。將方程(1)代入zt的定義得到:k=z11-k1-=z(4)將方程(4)代入xt的定義,得到:ck-=xz(5)使用方程(4),考慮zt=k(t)y(t)的時間導數,得到:z=1-k-k(6)從正文的方程(2.25)知道,k=k-c-n+gk,方程(6)可表示成:z=1-k-k-c-n+gk(7)為簡化上式,將方程(4)和方程(5)代入上式,得到:z=1-1-xz-n+gz(8)現在,對數化xt=c(t)k(t),考慮其時間導數,得到:xx=cc-kk(9)根據正文的方程(2.24)和方程(2.25),上式可表示成:xx=k-1-g+-k+c+n+gkk(10

18、)將方程(4)和方程(5)代入上式,再利用=得到:xx=x+n-(11)(d)(i)根據x為常量的假設,方程(8)可表示成z=1-1-n+g+x*z(12)為確定z的變化路徑,考慮方程(12)方程(12)為線性非齊次常微分方程。該方程的解包括通解zc和zp。簡單地設=1-n+g+x*。為求通解,考慮相應的齊次方程z+z=0,A1是積分常數,求解zz的微分方程得到通解:zc=A1e-t(13)為求特解,考慮非齊次方程z+z=1-,A2是積分常數,利用積分因子得到特解:zp=1-+A2e-t(14)因此,方程(12)的zc解表示成z=zc+zp=1-+A1+A2e-t(15)利用初始條件,z0替換

19、A1+A2,得到:z=1-+z0-1-e-t(16)為簡化1-,使用方程(2)和(3)消去x*,利用方程(4)得到:z=z*+e-tz0-z*(17)(ii) 可將方程(4)代入方程(1),求解y的路徑。由于已經得到z的路徑,將z的路徑代入方程(17),得到:y=z*+e-tz0-z*1-(18)使用k表示z的方程(4),上式可表示成:y=k*1-+e-tk01-k*1-1-(19)現在,分析經濟趨向平衡增長路徑的收斂速度是否不變。方程(19)兩端同減方程(2)確定的平衡增長路徑y(tǒng)*=k*=+g1-,再取對數求導:lny-y*=lnz1-+g1-(20)考慮上式的時間導數,得到:lny-y*t

20、=1-z1-1zz1-+g1-(21)上式顯然不是常數,收斂速度也不是常數。(e)需要知道正文的方程(2.24)和方程(2.25),或cc=k-1-g和k=k-c-(n+g)是否成立。使用方程(2.24)和方程(2.25)求解x/x,x/x成立的充要條件是方程(2.24)和方程(2.25)成立。已經方程(2.25)成立,以前使用該方程求解z。因此xx=0的充分必要條件是cc=0。假設xx=0,根據方程(11)可得:x*=-n(22)根據(a)題和(b)題,在平衡增長路徑上,x*等于+gg-(n+g),方程(22)可表示成:x*=-n(23)上式等同于方程(11),方程(2.24)和方程(2.2

21、5)得以成立。2.10拉姆塞-卡斯-庫普曼斯模型中的資本稅??紤]處于平衡增長路徑上的拉姆塞-卡斯-庫普曼斯經濟。假設在某一時刻(我們稱作0時),政府采取了一項對投資所得按稅率征稅的政策,因此家庭面臨的實際利率變?yōu)閞t=1-fkt。假設政府將稅收收入以一次性轉移支付的形式返還給家庭。最后,假設稅收政策是意料之外的。(a)該稅收政策如何影響c=0和k=0線?(b)經濟在0時會對該稅收政策作出何種反應?0時之后的動態(tài)學又是如何?(c)c和k在新舊兩種平衡增長路徑上的值有何不同?(d)本小題基于巴羅、曼昆和薩拉伊馬丁Barro, Mankiv, and Sala-i-Martin,1995假設存在許多

22、與本題相同的國家,各國工人們的偏好相同,但各國間的投資收入稅率可以不同。假設各國都處于其平衡增長路徑。(i)證明平衡增長路徑上的儲蓄率y*-c*y*關于是遞減的。(ii)低、高k*、高儲蓄率國家的居民是否有動機向低儲蓄率國家投資?為什么?(e)(c)小題中的答案是否說明補貼投資(即讓0)并通過一次性稅收為補貼籌資的政策可以提高福利?為什么?(f)如果政府并不返還稅收收入,而是將其用于政府購買,(a)小題和(b)小題中的答案會如何變化?答:(a)由于資本的稅后報酬變?yōu)椋簉t=1-fkt,家庭將改變每單位有效勞動的消費增長率來實現一生效用的最大化,即:ctct=1-fkt-g(1)在平衡增長路徑上

23、,c=0要求1-fkt=+g,即稅后報酬率為+g。為保持c=0,fkt必須上升,又因為fk,所以資本存量必須下降。因此,c=0這條曲線將會左移,如圖2-8所示。圖2-8 對投資增稅的影響家庭的每單位有效勞動的資本的歐拉方程仍為:kt=fkt-ct-n+gkt(2)由于政府將由這種稅收征集的收入又通過總量性轉移支出返還給家庭,所以家庭投資決策不受影響,因而k=0的軌跡不變。(b)在0時刻,由于資本的存量由歷史上的投資決策所決定,因而資本不會發(fā)生非連續(xù)的變化。資本仍然保持在原來的平衡增長路徑上的k*處。在0時刻,與每單位有效勞動的資本相反,每單位有效勞動的消費會由于征稅而立刻發(fā)生變化。由于稅收政策

24、的這種變化是非預期性的并且是毫無準備的,因此消費的變化是非連續(xù)的。由于政府的這種稅收征集,儲蓄和資本積累的回報會比以前低,家庭會轉而減少儲蓄,增加消費,在圖2-8上表現為c向上移動到A點,然后沿著新的均衡路徑移動。經濟沿著新的鞍點均衡路徑緩慢移動,最終移動到新的均衡點Enew。(c)由圖2-8可知,由于稅收扭曲了經濟刺激,因此稅后處在新的平衡增長路徑上的c與k的值將變小。(d)(i)由上述的分析可以看出,稅率越高,在平衡增長路徑上的k*越小,而且c=0曲線向左移動得越多,因而有k*0。在平衡增長路徑上,儲蓄率可以表示為:fk*-c*fk*,同時,k=0時,由kt=fkt-ct-n+gkt可以推

25、出fk*-c*=n+gk*,由此可以將儲蓄率表示為:s=n+gk*fk*(3)對方程(3)兩邊求關于稅率的導數:s=n+gk*fk*-n+gk*fk*k*fk*2可以簡化為:s=n+gfk*k*-n+gfk*k*fk*fk*k*=n+gfk*k*1-k*fk*fk*由于資本的收入份額為k*fk*fk*=Kk*,以及k*0,可以改寫上式為:s=n+gfk*k*1-Kk*0(4)以上便證明了平衡增長路徑上的儲蓄率y*-c*y*關于是遞減的。(ii)在低稅率、高資本存量和高儲蓄的國家的公民沒有動力去投資于低儲蓄的國家。由(a)可知,在平衡增長路徑上c=0,可以推出1-fkt=+g,即稅后的資本回報為

26、+g,假定在國家之間偏好與技術特征是相同的。因而在低儲蓄國家資本的稅后回報與高儲蓄國家的資本的稅后回報相同。因此,在低稅率、高資本存量和高儲蓄的國家的公民沒有動力去投資于低儲蓄的國家。(e)補貼投資不會增加福利。原先的市場結果便已經是中央計劃者能夠達到的社會效用最大化水平了,它給予了家庭最高可能的終生效用水平。從初始的E點開始,投資補貼能夠使消費短期內下降到A點,但最終經濟會沿著新的平衡增長路徑達到更大的消費水平Enew點??梢园l(fā)現短期的效用損失會超過長期的效用收益(都用現值形式表示),如圖2-9所示。圖2-9 對投資補貼不會增加福利(f)假定政府未將稅收所得返給家庭,而是用于政府購買。令Gt

27、為每單位有效勞動的政府購買,則每單位有效勞動的資本存量變化的歐拉方程仍為:kt=fkt-ct-Gt-n+gkt(5)政府購買被視為是政府的消費而不是投資,這將不會增加資本存量。由(5)可得,k=0曲線將向下移動。如圖2-10所示。由(a)可知,由于政府征稅,c=0曲線向左移動,k*移動到knew*,在新的平衡增長路徑上,每單位有效勞動的消費會低于存在政府的總量稅返還的情況。如圖2-10所示。圖2-10 稅收全部用于政府購買對經濟的影響2.11應用相圖分析預期變化的影響。考慮習題2.10中提到的政策,假設政府并不是在0時宣布并執(zhí)行該政策,而是在0時宣布將在以后某一時刻t1對投資收入按照稅率征稅。

28、(a)用相圖畫出t1之后c和k的動態(tài)學。(b)c在t1時刻的變化是否連續(xù)?為什么?(c)用相圖畫出t1之前c和k的動態(tài)學。(d)根據(a)、(b)和(c)的答案,c在0時應如何變化?(e)總結上述4個小問題,并把c和k的路徑描繪為時間的函數。答:(a)-(c)在開始征稅的時間t1之前,描述經濟動態(tài)變化的方程為:ctct=fkt-g(1)kt=fkt-ct-n+gkt(2)對于方程(1),在平衡增長路徑上,c=0可以推出fk=+g。由于政府返還總量稅,資本積累方程不受影響。在t1時刻征稅之后,c的歐拉方程為:ctct=1-fkt-g(3)在平衡增長路徑上,c=0可以推出1-fkt=+g,即稅后的

29、回報為+g。因此,稅前的資本回報fkt高于稅后的資本回報。為保持c=0,fkt必須上升,從而k必須下降。因此,在t1時刻,c=0曲線必須向左移動。如圖2-11所示。圖2-11 t1時刻征稅使得c=0向左移動不過值得注意的是,資本的動態(tài)在實際征稅之前仍由原先的歐拉方程決定。在t1時刻征稅之后,消費c不可能發(fā)生不連續(xù)的變化,原因在于家庭已經在事先知道了將要征稅的消息,家庭希望平滑消費。(d)在t1時刻征稅之后,消費不可能發(fā)生不連續(xù)的變化,同時經濟會達到新的平衡增長路徑。在0時刻宣布并施行征稅后,c會立即由原先的均衡點E移動到平衡增長路徑上的A點,如圖2-12所示。圖2-12 征稅對c=0曲線的影響

30、在A點,由于消費c太高,從而不足以將資本維持在原先的資本水平k*上,因此k開始下降。從0時刻到t1時刻,動態(tài)系統(tǒng)仍由原先的c=0的歐拉方程決定。消費在鞍點路徑之左,因此消費開始上升。在t1時刻經濟恰好移動到新的鞍點路徑,此時稅收開始執(zhí)行,并且動態(tài)系統(tǒng)仍由新的c=0的歐拉方程決定。因此,c開始下降,經濟最終移動到新的鞍點Enew。(e)每單位有效勞動的消費與每單位有效勞動的資本如圖2-13所示。(1)每單位有效勞動的消費的圖示(2)每單位有效勞動的資本的圖示圖2-13 每單位有效勞動的消費、有效勞動的資本圖示2.12應用相圖分析暫時性變化的影響??紤]習題2.11的如下兩種變形:(a)在0時刻,政

31、府宣布將對0時到其后某一時刻t1間的投資收入按照稅率征稅,而此后投資收入仍將免稅。(b)在0時,政府宣布將對t1時其后某一時刻t2間的投資收入按照稅率征稅,而t1之前和t2之后的投資收入仍將免稅。答:(a)第一問是分析預期到的稅收將在t1時刻結束,因而消費在t1時刻將不會發(fā)生非連續(xù)的變化。原因在于家庭的跨期消費最優(yōu)化要求家庭平滑消費。因此,在經濟返回到舊的鞍點路徑時,消費必須在t1時刻位于舊的鞍點路徑上。在征稅之前,即到0時刻,和在結束征稅之后,即t1時刻之后,經濟動態(tài)變化由下面兩個歐拉方程決定:ctct=fkt-g(1)kt=fkt-ct-n+gkt(2)資本積累的動態(tài)方程k=0不會受到征稅

32、的影響,但是,消費的動態(tài)方程c=0則會受到征稅的影響。在0時刻到t1時刻,資本的稅后回報為1-fkt=+g,為了保證c=0成立,fkt必須上升,由于fk0或lnkt1/1-時,上升意味著kt+1函數應該向上移動,相反,當lnkt0,和2kt+1kt2=sfkt1+g1+n0檢驗稻田條件:limk0kt+1kt=limkkt+1kt=1-1+g1+nrt+1,儲蓄會下降更快,相反,儲蓄則會下降的較慢。定義Zt=2+1+rt+1-1+rt+1-n2+1+rt+1,并將其代入(7),可得:St=12+Awt-ZtT(8)由于t+1期的資本存量等于t期的儲蓄,因此有:Kt+1=StL(9)將方程(9)

33、轉化為每單位有效勞動的形式,并使用方程(8),可得:kt+1=11+n12+wt-ZtTA(10)對于柯布道格拉斯生產函數,實際工資為:wt=1-kt(11)將方程(11)代入(10),產生新的關系,即:kt+1=11+n12+1-kt-ZtTA(12)(ii)為判斷社會保障的引入對平衡增長路徑上的k值的影響,必須判定Zt的正負號。如果Zt為正數,則社會保障稅T的引入會降低kt+1曲線并且降低k值。下面計算Zt:Zt=2+1+rt+1-1+rt+1-n2+1+rt+1=1+1+1+rt+1-1+rt+1-n2+1+rt+1上式可簡化為:Zt=1+rt+1+1+1+rt+1-rt+1-n2+1+

34、rt+1=1+rt+1+1+1+n2+1+rt+10因此,kt+1曲線向下移動,k*也降低了。(iii)如果經濟是初始動態(tài)有效的,則T的邊際增長會提高老年人的福利,但是它將使k*低于黃金律所要求的資本水平kGR,從而使未來一代人的福利惡化,降低他們的消費水平。但是,如果經濟初始是動態(tài)無效的,則k*kGR,則T的邊際增加會提高老年人的福利。同時,還會提高未來一代人的福利水平,從而是福利改進的。此時社會保障稅T的引入會降低過度的資本積累,從而消除動態(tài)無效率。(b)(i)方程(3),即第二期的預算約束變?yōu)椋篊2,t+1=1+rt+1St+1+rt+1T(13)從個人角度講,社會保障的回報率等于儲蓄的

35、回報率。由方程(13)推出儲蓄,即:St=C2,t+11+rt+1-T(14)將方程(14)代入(2)(即第一期的預算約束),可得: C1,t+C2,t+11+rt+1=Awt-T+T上式進一步簡化為:C1,t+C2,t+11+rt+1=Awt(15)家庭的最優(yōu)化行為產生了通常的歐拉方程,即:C2,t+1=11+1+rt+1C1,t將上式代入方程(15)中,可得:C1,t=1+2+Awt(16)為得到每個人的儲蓄,將方程(16)代入(2)中,可得:St=Awt-1+2+Awt-T上式進一步簡化為:St=12+Awt-T(17)社會保障的引入引起儲蓄的一對一的減少。t+1期的資本存量等于個人在t

36、期的儲蓄加上政府投資,即:Kt+1=StLt+TLt(18)將方程(18)轉化為每單位有效勞動的形式,并利用方程(17),可得:kt+1=11+n12+wt-TA+11+nTA上式進一步簡化為:kt+1=11+n12+wt利用方程(11)來替代上式中的工資,即:kt+1=11+n12+1-kt(19)因此,全額融資的社會保障稅T的引入對后續(xù)各期的資本沒有影響。(ii)因為全額融資的社會保障的引入對各期資本之間的關系沒有影響,因此在平衡增長路徑上各期資本是一樣的。各期的總資本與總儲蓄是一樣的,政府的作用僅僅是使年輕人儲蓄。因為社會保障回報率與儲蓄利率是一致的,因此來說個人對說誰來為他們儲蓄是無差

37、異的,個人將一對一的抵消政府為他們所做的任何儲蓄。2.18基本的世代交疊模型(本題來源于薩繆爾森,1958;阿萊,1947)。與戴蒙德模型類似,假設在t期出生的Lt個人只存活兩期,并且Lt=1+nLt-1。為了簡單起見,令效用函數為不折現的對數效用函數:Ut=lnC1,t+lnC2,t+1。在經濟的生產方面,該模型也比戴蒙德模型更為簡單:經濟中只有一種產品,它既可以用于消費,也可用于儲存;每個在t期出生的人都擁有A單位該產品;儲存每單位產品可使經濟主體在下一期得到x(x0)單位產品。最后。假設在最初的第0期,除L0個擁有A單位產品的年輕人之外,還有11+nL0個只生活在第0期的老年人,每個老年

38、人擁有Z單位的產品, 其效用就是在起始期的消費C2,0。(a)說明該經濟的分散化均衡。(提示:給定上述世代交疊的結構,某一代的成員是否會與另一代的成員進行交易?)(b)假設經濟人的稟賦中用于儲蓄的份額ft是不隨時間變化的常數,在這樣的路徑上請把人均總消費(總消費指所有年輕人與所有老年人的消費總和)表示為f的函數形式。如果x1+n,滿足0f1且最大化人均消費的f值是多少?分散均衡在這種情況下是否是帕累托有效的?如果不是,社會計劃者怎樣才能提高福利?答:(a)首先,該模型不存在任何一代的成員將會同另一代的成員交易的可能性。原因是即使年輕人愿意交換,但他們的交易對象只能是老人,而老人則因為下一期已去

39、世而不可能同年輕人進行交換。個人的效用函數為:Ut=lnC1,t+lnC2,t+1(1)預算約束為:C1,t+Ft=A(2)C2,t+1=xF(3)其中Ft是個人在第一期的儲蓄。將方程(3)代入(2),求個人的跨期預算約束:C1,t+C2,t+1x=A(4)用拉格朗日方法聯立方程(1)和(4)以求解個人終生效用的最大化,如下:L=lnC1,t+lnC2,t+1+A-C1,t+C2,t+1x求一階條件:LC1,t=1C1,t-=01C1,t=(5)LC2,t+1=1C2,t+1-x=01C2,t+1=x(6)將方程(5)代入(6),可得:C2,t+1=xC1,t(7)將方程(7)代入跨期預算約束

40、式(4)中,可得:C1,t+xC1,t=A,上式可簡化為:C1,t=A2(8)求第二期的消費,將方程(8)代入(7)中,可得:C2,t+1=xA2(9)當年輕人將他的財富一半儲蓄時,下一期他可以消費xA2。由于是對數效用函數,因此,個人將其稟賦儲蓄的比例并不依賴于儲蓄的回報率。(b)在t時刻的總消費為:Ct=C1,tLt+=C2,tLt-1其中,Lt是年輕人的數量,Lt-1是老年人的數量。每個年輕人消費他的稟賦的一部分1-fA,每個老年人消費他的稟賦的總回報fxA。由式Ct=1-fALt+fxALt-1,對其兩邊除以ALt以轉化為每單位有效勞動的形式,即:CtALt=1-f+fx/1+n因此,

41、每單位有效勞動的消費為一加權和。因為x1+n,所以,當權數為1時,每單位有效勞動的消費為一加權和達到最大化。因此,分散化均衡(即權數為)不是帕累托有效的。因為跨代交易是不可能的,因此個人儲蓄為年老時提供消費,即使儲蓄的回報率非常低,他們也必須這樣做。但是在一個集權經濟中,社會計劃者則可以從年輕人手中取走一單位物品而給每個老年人1+n的物品。由于x1+n,所以得到了一個更高的回報率。因此,社會計劃者可以從年輕人手中取走他們一半的財富交給老年人去消費,從而提高社會的總體福利。社會計劃者可以每期都這么做,即允許個人在年輕時消費A2,而在年老時消費1+nA2,高于在分散經濟的情況下每個人在年老時消費的

42、xA2。2.19薩繆爾森世代交疊模型中的穩(wěn)態(tài)貨幣均衡(本題來源于薩繆爾森,1958)??紤]習題2.18中的設定,假設xPtPt+1他將消費一半的稟賦,存儲剩余的一半而不持有任何貨幣,因為貨幣的回報率低于儲蓄的回報率。因此有:C1,t=A2, Ft=A2, MtdPt=0, C2,t+1=xA2情況2:xPtPt+1他將用貨幣持有一半的稟賦,即他將消費一半的財富而賣掉另一半的稟賦。因此有:C1,t=A2, Ft=0, MtdPt=A2, C2,t+1=PtPt+1A2情況3:x=PtPt+1由于貨幣和存儲帶來同樣的回報,因此他將消費一半的稟賦,對于另一半,則在貨幣和存儲兩者之間無差異。令0,1為

43、以貨幣形式持有的比例。因此有:C1,t=A2, Ft=1-A2, MtdPt=A2, C2,t+1=PtPt+1A2(b)均衡要求總的實際貨幣需求等于總的實際貨幣供給??倢嶋H貨幣需求=LtA2總實際貨幣供給=L01+nMPt=Lt1+nt+1MPt在上式中,在0時刻,每個老人擁有M單位貨幣,共有L01+n個老人。最后一步用了Lt=1+ntL0,從而有L0=Lt1+nt。聯立總實際貨幣需求和總實際貨幣供給兩個公式,可得:LtA2=Lt1+nt+1MPtPt=2MA1+nt(5)因此有:總實際貨幣需求=Lt+1A2=1+nLtA2總實際貨幣供給=Lt1+nt+1MPt+1下面使用均衡條件求Pt+1

44、,即:1+nL0A2=Lt1+nt+1MPt+1Pt+1=2MA1+nt+2(6)用方程(6)除以(5),有:Pt+1Pt=11+nPt+1=Pt1+n上面的分析對任何的t0成立,因此Pt+1=Pt1+n是一個均衡。這表明如果貨幣被引入到一個動態(tài)無效率的經濟中,個人將不會選擇存儲。(c)由于Pt+1=Ptx,所以貨幣的回報等于存儲的回報。此時,個人對于以何種形式持有稟賦是無差異的。令0,1為儲蓄中以貨幣形式持有的比例。t期的總實際貨幣需求和總實際貨幣供給的表達式如下:總實際貨幣需求=LttA2總實際貨幣供給=L01+nMPt=Lt1+nt+1MPt使用均衡條件求Pt:LttA2=Lt1+nt+

45、1MPtPt=2MtA1+nt(7)t+1期的總實際貨幣需求和總實際貨幣供給的表達式如下:總實際貨幣需求=Lt+1t+1A2=1+nLtt+1A2總實際貨幣供給=Lt1+nt+1MPt+1使用均衡條件求Pt+1:1+nLtt+1A2=Lt1+nt+1MPt+1(8)用方程(8)除以(7),可得:Pt+1Pt=tt+111+n因為Pt+1Pt=1x,所以有:tt+111+n=1xtt+1=x1+n0因此對于所有的t0,Pt+1=Ptx=將是任何滿足tt+1=x1+n的的路徑的一個均衡。(d)Pt=代表貨幣是無價值的,也是一種均衡。這種情況是因為年輕人相信貨幣在下一期是無價值的,因此這一代人將不會

46、接受貨幣作為儲存的替代物。在這種情況下,年輕人消費稟賦的一半然后儲存另一半,而老年人則擁有一堆無價值的貨幣。這時,總實際貨幣需求與總實際貨幣供給相等且都是0。如果沒有人相信下一代人將接受貨幣作為存儲的替代物,這種均衡將持續(xù)到未來各期。在T期,這種情況將是唯一的均衡。在T期沒有年輕人愿意出賣稟賦以換取貨幣。年輕人將通過消費所有的存儲來最大化一生的效用,老年人將持有一堆毫無價值的貨幣。因此,在T-1期,老年人因為知道下一期貨幣毫無價值,沒有人愿意出賣稟賦以換取貨幣。T-1期將沒有人愿意持有貨幣,逆向歸納,將沒有人愿意在任何一期出賣稟賦來換取貨幣。2.20動態(tài)無效率的來源。(本題來自于謝爾,1971

47、。)戴蒙德模型和薩繆爾森模型可以在下面兩個方面進行改變:第一,不完全市場,即由于一個人不能與尚未出生的人交易,從而排除了一些可能的交易;第二,無限時期,即由于時間是無窮無盡的,因此存在無窮數量的經濟人。本題試圖討論其中哪一方面是導致動態(tài)無效率的可能原因。為了簡單起見,我們著重討論薩繆爾森世代交疊模型(參見前面兩題),假設對數效用并且不考慮折現。為簡化本題,進一步假設n=0以及0 x0,Qt+1=Qtx等價于x=QtQt+1。換句話說,存儲的回報率等價于交易的回報率,因此個人對于存儲和交易是無差異的。令t0,1代表儲蓄的份額A2中被賣掉的部分。因此個人在t期賣掉tA2。這允許個人在t+1期年老時

48、購買tQtQt+1A2。個人存儲他的儲蓄的1-t,有下式:St=1-tA2(3)在t+1期的消費將等于個人購買的數量加上存儲的數量,即:C2,t+1=tQtQt+1A2+1-txA2(4)考慮Qt+1=Qtx的情況,方程(4)可以寫為:C2,t+1=txA2+1-txA2=xA2(5)考慮在任意的t+1期,令L代表各期總人數,它是不變的??偣┙o等于L乘以個人想賣掉的數量t+1A2。則總供給為:yt+1=Lt+1A2(6)總需求為t+1期總的老人數量L乘以個人想要購買的數量(tQtQt+1A2??傂枨鬄椋篸t+1=LQtQt+1tA2(7)要達到市場出清,總需求等于總供給,即:Lt+1A2=LQ

49、tQt+1tA2該式簡化為:t+1=QtQt+1t(8)由于均衡的價格路徑為:Qt+1=Qtx,方程(8)給出的均衡條件可以寫為:t+1=xt(9)考慮0期的情況,老年人僅僅消費他們的稟賦,因此在0期令0=0以實現市場出清,則方程(9)表明對于任何的t0,都有t=0。均衡的結果與習題2.17的結果是一致的。個人在第一期消費一半的稟賦而存儲另一半,在第二期消費xA2。由于x1+n,(因為n=0和x1),因此是動態(tài)無效率的。因此,通過在開始之前允許交易以消除市場的不完全性并不能消除動態(tài)無效率。(ii)假定拍賣者宣布Qt+1Qtx或者xQtx或者 xQtQt+1,這意味著存儲占優(yōu)于交易,即在t期年輕人將存儲所有的儲蓄,并且購買A2。對于t期的老人來講,Qt+1是無關的,老人基于在年老時QtQt+1=x來做決策。因此對于老人來說,正如第一部分分析的,

溫馨提示

  • 1. 本站所有資源如無特殊說明,都需要本地電腦安裝OFFICE2007和PDF閱讀器。圖紙軟件為CAD,CAXA,PROE,UG,SolidWorks等.壓縮文件請下載最新的WinRAR軟件解壓。
  • 2. 本站的文檔不包含任何第三方提供的附件圖紙等,如果需要附件,請聯系上傳者。文件的所有權益歸上傳用戶所有。
  • 3. 本站RAR壓縮包中若帶圖紙,網頁內容里面會有圖紙預覽,若沒有圖紙預覽就沒有圖紙。
  • 4. 未經權益所有人同意不得將文件中的內容挪作商業(yè)或盈利用途。
  • 5. 人人文庫網僅提供信息存儲空間,僅對用戶上傳內容的表現方式做保護處理,對用戶上傳分享的文檔內容本身不做任何修改或編輯,并不能對任何下載內容負責。
  • 6. 下載文件中如有侵權或不適當內容,請與我們聯系,我們立即糾正。
  • 7. 本站不保證下載資源的準確性、安全性和完整性, 同時也不承擔用戶因使用這些下載資源對自己和他人造成任何形式的傷害或損失。

評論

0/150

提交評論